beam with end moment

Solution to Problem 696-697 | Beam Deflection by Method of Superposition

Problem 696
In Fig. P-696, determine the value of P for which the deflection under P will be zero.
 

Solution to Problem 692 | Beam Deflection by Method of Superposition

Problem 692
Find the value of EIδ midway between the supports for the beam shown in Fig. P-692. (Hint: Combine Case No. 11 and one half of Case No. 8.)
 

Solution to Problem 689 | Beam Deflection by Method of Superposition

Problem 689
The beam shown in Fig. P-689 has a rectangular cross section 4 inches wide by 8 inches deep. Compute the value of P that will limit the midspan deflection to 0.5 inch. Use E = 1.5 × 106 psi.

Solution to Problem 688 | Beam Deflection by Method of Superposition

Problem 688
Determine the midspan value of EIδ at the left end of the beam shown in Fig. P-688.
 

Subscribe to RSS - beam with end moment